BTW I might add one thing.
Even if a magnetic field can exceed the speed of light in this sense it is
not really clear how that compares to any other form of movement.

For instance a magnetic field can be made to appear to rotate by turning on
electromagnets  in order as with the rotating stator fields in synchronous
motors, plausibly such could be made to appear to rotate faster than C but
it would not be real motion.

Here too IF it even rotates it is not clear if it can be considered the same
as motion of a particle or wave.


On Wed, Jun 10, 2009 at 2:31 PM, John Berry <aethe...@gmail.com> wrote:

> Ok, well if it's axial field orientation then I would say you have walked
> right into the N-Machine paradox.
>
> Which is, does the magnetic field actually rotate when you rotate a magnet
> on an axial orientation?
>
> It is very difficult to prove since the only effect from an axially
> rotating magnet is a generally very weak electric field gradient.
>
> For instance if you rotate a conductive magnet and tap the voltage
> difference between the shaft and the periphery you get a voltage but is that
> a voltage induced in the magnet due to the field not rotating or is it a
> voltage induced in the external circuit?
>
> Now let's try something that could be more easily measured if more
> preposterous when it comes to actual practical implementation. (but that is
> why they are called thought experiments)
>
> Lets have a magnet that accelerates very fast  in one direction, stops and
> reverses direction, wash rinse and repeat.
>
> Now we have a bit of a dilemma since the magnetic field IF it rotates with
> the magnet should now be inducing a time varying electric field of alternate
> polarity, furthermore this voltage change/reversal will radiate out from the
> magnet seemingly at the speed of light, however this creates many issues,.
>
> The most obvious of which is that the direction of rotation of the flux
> inside the magnet will differ to the direction of rotation of flux outside
> the magnet at a distance! (unless the speed of light is breached)
>
> Radiation however is hard to imagine with an axially oriented magnetic
> field.
>
> At some distance the field IF it is rotating (which we can verify with the
> above experiment) will be moving at a speed greater than C or seemingly
> should.
>
> To make this semi practical (provided you have proven the field rotates)
> you would want a very very long, say mile long neodymium magnet encased in
> some seriously hardy material, get it up to speed and thanks to the distance
> between the poles the field should radiate out from the magnet a good
> distance. (alternately a magnetic steel with a stationary electromagnet
> around it)
>
> I can't be bothered to do the math but I'd bet that within extreme but
> achievable rpm's and by measuring the field as far away as possible you
> could exceed light speed with todays technology.
>
> So then either C will be exceeded, the frame of space will drag and or the
> field will become shielded or no longer supported by space.
>
> The same essential experiment could be achieved by rotating an
> electrostatically charged object.
>
> But again it is quite possible that the first does not rotate with the
> source of the field.
>
> Hooper's Motional E-field work comes the closest to answering that riddle.
>
>
>
>
>
>
>
> On Wed, Jun 10, 2009 at 1:45 PM, Michael Crosiar <crosia...@yahoo.com>wrote:
>
>>  >Ok, so take a magnet (it's a thought experiment so the realities of
>> near relativistic speeds of a spinning object interest me not),
>>
>> Thank you!
>>
>> >rotate it in such a way that it's magnetic poles flip.
>>
>> Actually, I don't think this is the normal rotation, I am more interested
>> in the axial orientation. I see the problem with the "pole flip" as being
>> the stability caused by the E-Field - I suspect strongly it would radiate.
>> But I could be wrong.
>>
>> >The field at some distance from the magnet must logically be moving
>> greater than C.
>>
>> That is the point I wanted to get too!!!! Yes, but nothing can go faster
>> than C, so something must give.
>>
>> >So we have 2 possibilities.
>> >First is that the field will entrain the aether and drag it (frame
>> dragging) and hence the field will not be moving at a speed greater than C.
>> (though the field may be in effect shielded from expanding beyond a point
>> where the moving >aether ceases to be entrained by the field)
>>
>> >To what degree that answer would be acceptable to anyone besides me I'm
>> not totally sure since I have not bothered to understand what frame dragging
>> really refers to as IMO special relativity is wrong and science made a wrong
>> >turn when it disregarded the aether, and everyone considered to have
>> 'disproven' the aether including Enstein and MM still believed it existed.
>>
>> >The other possibility is that, and this one sounds possibly more likely,
>> the fields will be thrown off as radiation.
>>
>> Now this I disagree with, these are not the only possibilities. The other
>> possibility is that delta-t changes - C remains the same because the rate at
>> which time moves forward is changed - the measuring stick and the clock are
>> now in a different frame of reference and must be looked at just like with
>> the spaceship and spacestation example discussed before. Due to the rotation
>> this is no longer an inertial frame of reference. If this is correct, there
>> a certain predictions we can make. First, due to the rotation of the field,
>> it should acquire inertial mass- attempts to move the field (and that which
>> is generating it) should resist being moved. Next, time within the field
>> should run at a different rate, either slower or faster depending on the
>> direction of rotation relative to the n/s pole (Ok, I have not shown this
>> yet...). This could be tested by placing a radioactive isotope inside the
>> field and measuring the radiation compared to a control sample outside the
>> field. Time measurements would also be affected. And, if I am right, any
>> mass contained by the field should gain or loose its gravitational mass
>> (actually, I'm way over-simplifying here). I would also expect a strong
>> E-Field from the outer most part of the field to the axis of rotation, as
>> would be normally expected. I strongly suspect that these relativistic
>> effects will have direction and magnitude.
>>
>> >Fields don't need to expand and shrink IMO to be thrown off as
>> radiatiuon, it merely requires a swift enough change that causes the more
>> distant part of a field to decouple from the source that generated it.
>>
>> I'm not sure about this one. I do suspect that attempting to change the
>> angular velocity to rapidly would cause the field to rotate or "vibrate" at
>> harmonic frequencies in addition to the primary frequency which would likely
>> cause EM radiation.
>>
>> >Of course this has an implication, that the magnetic field would put a
>> drag on it because it can't freely radiate energy right?
>>
>> >Would not some pulsars or something else astronomical be in the range of
>> powerful enough to be an astronomical version of this experiment?
>>
>> Maybe, but I'm also thinking of the EM fields that exist in atoms, shells
>> and electrons...
>>
>> >Or would their magnetic field be more likely to be in an axial
>> orientation?  I guess so? or not?
>>
>> I pretty sure, but I will need to think about it. I think the issue has to
>> do with the symmetry of the E-field, or lack there of.
>>
>> >So no astronomical versions of this thought experiment?
>> I would think pulsars and black holes, but don't we already acknowledge
>> relativistic effects for these objects?
>> C. Michael Crosiar
>>
>>
>> On Wed, Jun 10, 2009 at 7:27 AM, OrionWorks <svj.orionwo...@gmail.com>wrote:
>>
>>> From Michael Corsiar:
>>>
>>> > What makes you believe it would radiate any EM?
>>>
>>> I don't. Not sure what to believe. It's why I'm askin...
>>>
>>> > The field is rotating, it is not expanding or collapsing.
>>> > I see this as a standing or scalar wave. I would expect
>>> > an E-field, but no EM radiation.
>>>
>>> I think the reason I have speculated that there might possibly be EM
>>> radiaion generated is that if the PM was positioned in such a manner
>>> that the opposite poles were swinging 90 degrees in relation to the
>>> rotational axis I would assume that there would be a lot of dynamic
>>> magnetic changes occuring. Seems to me that if one placed a circular
>>> wire close to the rotating PM, would not the circular wire be
>>> influenced by the rotating PM causing some level of AC to course
>>> through the wire?
>>>
>>> I think my ignorance stems from the fact that it's not clear to me
>>> what the crucial differences are between standing or scaler waves and
>>> EM radiation.
>>>
>>> Regards
>>> Steven Vincent Johnson
>>> www.OrionWorks.com
>>> www.zazzle.com/orionworks
>>>
>>>
>>
>>
>

Reply via email to